Divergence of electrostatic field?

Click For Summary
The discussion addresses the divergence of the electric field derived from Gauss' law and Coulomb's law. It highlights that the divergence calculation of the electric field is valid only for regions where the charge density is zero, specifically for r > 0. At the origin, the divergence behaves as a delta function, indicating an infinite value, which corresponds to the charge density at that point. This means that both the divergence and the charge density are treated as distributions rather than traditional functions. The key takeaway is the need to consider the behavior at the origin to reconcile the apparent contradiction in the divergence of the electric field.
terryds
Messages
392
Reaction score
13
Poster has been reminded to post clear images and scans (not dark fuzzy phone pictures)

Homework Statement



By Gauss' law, how is it able to obtain ## \nabla \cdot \vec{E} = \frac{\rho}{\epsilon_0} ## ?

By Coulomb's law, ##\vec{E} = \frac{1}{4\pi\epsilon_0} \frac{q}{r^2} \hat{r}##

I calculate the divergence of ##\frac{1}{r^2} \hat{r}## and get the result is zero

That means the divergence of ##\frac{1}{4\pi\epsilon_0} \frac{q}{r^2} \hat{r}## is also zero too.

How come it is ## \nabla \cdot \vec{E} = \frac{\rho}{\epsilon_0} ## ?

Homework Equations



Gauss' Law
Coulomb's Law

The Attempt at a Solution



11ukq45.jpg
[/B]
 
Physics news on Phys.org
Your computation is only valid for ##r > 0## where the charge density is zero. The divergence is a delta function at the origin, just as the charge density.
 
  • Like
Likes terryds
Orodruin said:
Your computation is only valid for ##r > 0## where the charge density is zero. The divergence is a delta function at the origin, just as the charge density.

So, what happens at point (0,0,0) ?The divergence is infinity or ##\frac{\rho}{\epsilon_0}##?
 
The divergence is ##q \delta^{(3)}(\vec x)/\epsilon_0## and so is the charge density/epsilon0. Both are "infinite" at the origin. (The proper treatment is to consider them distributions rather than functions)
 
  • Like
Likes terryds
Question: A clock's minute hand has length 4 and its hour hand has length 3. What is the distance between the tips at the moment when it is increasing most rapidly?(Putnam Exam Question) Answer: Making assumption that both the hands moves at constant angular velocities, the answer is ## \sqrt{7} .## But don't you think this assumption is somewhat doubtful and wrong?

Similar threads

  • · Replies 9 ·
Replies
9
Views
2K
  • · Replies 6 ·
Replies
6
Views
1K
  • · Replies 29 ·
Replies
29
Views
1K
  • · Replies 9 ·
Replies
9
Views
1K
  • · Replies 6 ·
Replies
6
Views
2K
  • · Replies 8 ·
Replies
8
Views
1K
Replies
11
Views
1K
  • · Replies 5 ·
Replies
5
Views
2K
  • · Replies 1 ·
Replies
1
Views
2K
  • · Replies 6 ·
Replies
6
Views
3K